LimSup of a Sequence: Existence & Unbounded Cases

  • Thread starter Thread starter C.E
  • Start date Start date
  • Tags Tags
    Analysis
Click For Summary
The discussion focuses on the definition and existence of the limit superior (limsup) of a sequence, defined as limsup_{n \to \infty} a_n = lim_{n \to \infty} sup_{k ≥ n} a_k. It is clarified that when a sequence is unbounded above, the limsup is infinite. The sequence v_n, defined as sup_{k ≥ n} a_k, is examined for its properties, particularly whether it is increasing or decreasing. It is noted that if v_n is bounded, its limit exists due to the completeness axiom, while if it is unbounded and monotonically decreasing, it approaches negative infinity. The discussion emphasizes the need for a detailed explanation of these concepts, especially regarding the behavior of v_n in different scenarios.
C.E
Messages
100
Reaction score
0
1. Let a_n be a sequence in R. Define \limsup_{n \to \infty} a_n. Explain why it exists including the cases when the sequence is unbounded.

3. I did the following, is it ok?

definition:

\limsup_{n \to \infty} a_n=\lim_{n \to \infty} sup_{k \geq n}a_k.

Is this right, I seem to have lost this part of my notes!

I know that when it is not bounded (above) the limsup is infinite. What I am really having

trouble with is explaining why it exists. What do you think they want me to say? (By the

way this question was worth 10 marks so they want a lot of detail).
 
Physics news on Phys.org
Let v_n=sup_{k \geq n}a_k

What kind of sequence is v_n? increasing, decreasing, bounded,...
 
Is v_n always a decreasing sequence? It may be bounded or it may not be. If it is bounded then by the completeness axiom its limit must exist, but what if it is not bounded?
 
Last edited:
C.E said:
Is v_n always a decreasing sequence? It may be bounded or it may not be. If it is bounded then by the completeness axiom its limit must exist, but what if it is not bounded?

If it's monotonically decreasing but unbounded, then for every positive B, there is an N such that

v_n \leq -B for all n \geq N

in which case by definition we write

\lim_{n \rightarrow \infty} v_n = -\infty
 
Is it increasing then? If so why?
 
Question: A clock's minute hand has length 4 and its hour hand has length 3. What is the distance between the tips at the moment when it is increasing most rapidly?(Putnam Exam Question) Answer: Making assumption that both the hands moves at constant angular velocities, the answer is ## \sqrt{7} .## But don't you think this assumption is somewhat doubtful and wrong?

Similar threads

  • · Replies 8 ·
Replies
8
Views
3K
  • · Replies 2 ·
Replies
2
Views
2K
  • · Replies 8 ·
Replies
8
Views
2K
  • · Replies 1 ·
Replies
1
Views
2K
  • · Replies 2 ·
Replies
2
Views
2K
  • · Replies 1 ·
Replies
1
Views
1K
  • · Replies 28 ·
Replies
28
Views
3K
Replies
5
Views
2K
  • · Replies 7 ·
Replies
7
Views
2K
  • · Replies 13 ·
Replies
13
Views
2K